หัวข้อ: True - False Marathon
ดูหนึ่งข้อความ
  #28  
Old 28 ธันวาคม 2006, 23:04
nongtum's Avatar
nongtum nongtum ไม่อยู่ในระบบ
ผู้พิทักษ์กฎทั่วไป
 
วันที่สมัครสมาชิก: 10 เมษายน 2005
ข้อความ: 3,246
nongtum is on a distinguished road
Post

อ้างอิง:
ข้อความเดิมของคุณ Mastermander:
35.$$\sqrt {2\sqrt {3\sqrt {4\sqrt {...} } } } < 3$$
ให้ $x=\sqrt {2\sqrt {3\sqrt {4\sqrt {...} } } }$ จะได้ $\displaystyle\ln x=\ln\prod_{n=2}^{\infty} n^{1/2^{n-1}}=\sum_{n=2}^{\infty}\frac{\ln n}{2^{n-1}}$
โดย ratio test จะได้ $\displaystyle\frac{\ln (n+1)/2^n}{\ln n/2^{n-1}}<1$ ผลรวมนี้จึงลู่เข้า แต่ผมยังนึกไม่ออกว่าจะหาผลรวมยังไงครับ
อ้างอิง:
ข้อความเดิมของคุณ Mastermander:
36. The equation $x^2+y^2+z^2=x^3+y^3+z^3$ has infinitely integer solutions.
ผิด เพราะเมื่อ $x,y,z>1$ จะได้ $\sum x^3>\sum x^2$
เมื่อ $0<x,y,z<1$ จะได้ $0<\sum x^3<\sum x^2$
เมื่อ $x,y,z<0$ จะได้ $\sum x^3<0<\sum x^2$
สมการนี้จึงมี $x=y=z=0$ และั $x=y=z=1$ เป็นคำตอบครับ
อ้างอิง:
ข้อความเดิมของคุณ nooonuii:
41. มีจำนวนเฉพาะ $p,q,r$ ที่แตกต่างกันซึ่งทำให้ $\displaystyle{\frac{p}{q}+\frac{q}{r}+\frac{r}{p}}$ เป็นจำนวนเต็ม
ผิด เพราะหากสมมติให้มีจำนวนเฉพาะ $p,q,r$ ที่แตกต่างกัน และมีจำนวนเต็ม $s$ ที่ $s=\displaystyle{\frac{p}{q}+\frac{q}{r}+\frac{r}{p}}=\frac{p^2r+q^2p+r^2q}{pqr}$
ดังนั้น $pqrs\equiv0\pmod p$ แต่ $pqrs=p^2r+q^2p+r^2q\equiv r^2q\not\equiv 0\pmod p$
__________________
คนไทยร่วมใจอย่าใช้ภาษาวิบัติ
ฝึกพิมพ์สัญลักษณ์สักนิด ชีวิต(คนตอบและคนถาม)จะง่ายขึ้นเยอะ (จริงๆนะ)

Stay Hungry. Stay Foolish.
ตอบพร้อมอ้างอิงข้อความนี้